Variance and covariance inequalityMinimizing the variance of weighted sum of two random variables with respect to the weightsFinding covariance given varianceVariance and covarianceVariance, Covariance, and Correlation answer checkConditional Expectation and Variance QuestionVariance and covariance between 2 variablesHow to prove an equality envolving variance and covarianceFinding the covariance of X,Y using variance?Covariance between given iid random variable and sample averageVariance of ratio of mean value of functions of a random variable

Why commonly or frequently used fonts sizes are even numbers like 10px, 12px, 16px, 24px, or 32px?

With today's technology, could iron be smelted at La Rinconada?

Is there any deeper thematic meaning to the white horse that Arya finds in The Bells (S08E05)?

Were any toxic metals used in the International Space Station?

Did any "washouts" of the Mercury program eventually become astronauts?

Is there an academic word that means "to split hairs over"?

Meaning of "legitimate" in Carl Jung's quote "Neurosis is always a substitute for legitimate suffering."

Wireless headphones interfere with Wi-Fi signal on laptop

Does it matter what way the tires go if no directional arrow?

Why did Varys remove his rings?

How did the horses get to space?

How to rename multiple files in a directory at the same time

Promotion comes with unexpected 24/7/365 on-call

Why would company (decision makers) wait for someone to retire, rather than lay them off, when their role is no longer needed?

What is the effect of the Feeblemind spell on Ability Score Improvements?

Why do OOK transmissions have bandwidth?

How does Ctrl+c and Ctrl+v work?

How could it be that 80% of townspeople were farmers during the Edo period in Japan?

What do the "optional" resistor and capacitor do in this circuit?

tikzcd diagram within an array

Understanding Python syntax in lists vs series

Can my American children re-enter the USA by International flight with a passport card? Being that their passport book has expired

Why is Drogon so much better in battle than Rhaegal and Viserion?

Assembly writer vs compiler



Variance and covariance inequality


Minimizing the variance of weighted sum of two random variables with respect to the weightsFinding covariance given varianceVariance and covarianceVariance, Covariance, and Correlation answer checkConditional Expectation and Variance QuestionVariance and covariance between 2 variablesHow to prove an equality envolving variance and covarianceFinding the covariance of X,Y using variance?Covariance between given iid random variable and sample averageVariance of ratio of mean value of functions of a random variable













1












$begingroup$


Given a real-valued random variable $X$, is



$$2mathbb E[X] mathrmVar(X) geq mathrmCov(X, X^2)$$



true?



Any pointers for how to tackle this problem would be immensely helpful.










share|cite|improve this question









$endgroup$











  • $begingroup$
    One trick is to write a little computer program to generate examples randomly, which you can use to find counterexamples. Such as $P(X=a)=p, P(X=b)=1-p$, where you use a random number generator to pick the parameters $a,b,p$ and exact formulas in terms of $a,b.p$ for the relevant moments of $X$.
    $endgroup$
    – kimchi lover
    4 hours ago










  • $begingroup$
    I expect that the answer will follow quickly after applying the well-known identities $textVar(X)=mathbb E[X^2]-mathbb E[X]^2$ and $textCov(X,Y)=mathbb E[XY]-mathbb E[X]mathbb E[Y]$
    $endgroup$
    – M. Nestor
    4 hours ago















1












$begingroup$


Given a real-valued random variable $X$, is



$$2mathbb E[X] mathrmVar(X) geq mathrmCov(X, X^2)$$



true?



Any pointers for how to tackle this problem would be immensely helpful.










share|cite|improve this question









$endgroup$











  • $begingroup$
    One trick is to write a little computer program to generate examples randomly, which you can use to find counterexamples. Such as $P(X=a)=p, P(X=b)=1-p$, where you use a random number generator to pick the parameters $a,b,p$ and exact formulas in terms of $a,b.p$ for the relevant moments of $X$.
    $endgroup$
    – kimchi lover
    4 hours ago










  • $begingroup$
    I expect that the answer will follow quickly after applying the well-known identities $textVar(X)=mathbb E[X^2]-mathbb E[X]^2$ and $textCov(X,Y)=mathbb E[XY]-mathbb E[X]mathbb E[Y]$
    $endgroup$
    – M. Nestor
    4 hours ago













1












1








1





$begingroup$


Given a real-valued random variable $X$, is



$$2mathbb E[X] mathrmVar(X) geq mathrmCov(X, X^2)$$



true?



Any pointers for how to tackle this problem would be immensely helpful.










share|cite|improve this question









$endgroup$




Given a real-valued random variable $X$, is



$$2mathbb E[X] mathrmVar(X) geq mathrmCov(X, X^2)$$



true?



Any pointers for how to tackle this problem would be immensely helpful.







probability inequality variance covariance






share|cite|improve this question













share|cite|improve this question











share|cite|improve this question




share|cite|improve this question










asked 4 hours ago









sk1ll3rsk1ll3r

405




405











  • $begingroup$
    One trick is to write a little computer program to generate examples randomly, which you can use to find counterexamples. Such as $P(X=a)=p, P(X=b)=1-p$, where you use a random number generator to pick the parameters $a,b,p$ and exact formulas in terms of $a,b.p$ for the relevant moments of $X$.
    $endgroup$
    – kimchi lover
    4 hours ago










  • $begingroup$
    I expect that the answer will follow quickly after applying the well-known identities $textVar(X)=mathbb E[X^2]-mathbb E[X]^2$ and $textCov(X,Y)=mathbb E[XY]-mathbb E[X]mathbb E[Y]$
    $endgroup$
    – M. Nestor
    4 hours ago
















  • $begingroup$
    One trick is to write a little computer program to generate examples randomly, which you can use to find counterexamples. Such as $P(X=a)=p, P(X=b)=1-p$, where you use a random number generator to pick the parameters $a,b,p$ and exact formulas in terms of $a,b.p$ for the relevant moments of $X$.
    $endgroup$
    – kimchi lover
    4 hours ago










  • $begingroup$
    I expect that the answer will follow quickly after applying the well-known identities $textVar(X)=mathbb E[X^2]-mathbb E[X]^2$ and $textCov(X,Y)=mathbb E[XY]-mathbb E[X]mathbb E[Y]$
    $endgroup$
    – M. Nestor
    4 hours ago















$begingroup$
One trick is to write a little computer program to generate examples randomly, which you can use to find counterexamples. Such as $P(X=a)=p, P(X=b)=1-p$, where you use a random number generator to pick the parameters $a,b,p$ and exact formulas in terms of $a,b.p$ for the relevant moments of $X$.
$endgroup$
– kimchi lover
4 hours ago




$begingroup$
One trick is to write a little computer program to generate examples randomly, which you can use to find counterexamples. Such as $P(X=a)=p, P(X=b)=1-p$, where you use a random number generator to pick the parameters $a,b,p$ and exact formulas in terms of $a,b.p$ for the relevant moments of $X$.
$endgroup$
– kimchi lover
4 hours ago












$begingroup$
I expect that the answer will follow quickly after applying the well-known identities $textVar(X)=mathbb E[X^2]-mathbb E[X]^2$ and $textCov(X,Y)=mathbb E[XY]-mathbb E[X]mathbb E[Y]$
$endgroup$
– M. Nestor
4 hours ago




$begingroup$
I expect that the answer will follow quickly after applying the well-known identities $textVar(X)=mathbb E[X^2]-mathbb E[X]^2$ and $textCov(X,Y)=mathbb E[XY]-mathbb E[X]mathbb E[Y]$
$endgroup$
– M. Nestor
4 hours ago










2 Answers
2






active

oldest

votes


















2












$begingroup$

Use the variance and covariance identities
$$textVar(X)=mathbb E[X^2] − mathbb E[X]^2$$
and
$$textCov(X,Y) = mathbb E[XY] − mathbb E[X] mathbb E[Y]$$



Then the given inequality is equivalent to



$$ 2 mathbb E[X] Big( mathbb E[X^2] - mathbb E[X]^2 Big)
overset?geq
mathbb E[X^3] - mathbb E[X] mathbb E[X^2] tag1$$



For a counterexample, let $X sim textExp(lambda)$ be the exponential distribution with parameter $lambda in (0, infty)$ and probability mass function



$$ f_X(x) = begincases
lambda e^-lambda x & textif x geq 0 \
0 & textif x < 0
endcases $$



Its $n$-th moment is given by $mathbb E[X^n]=fracn!lambda^n$, so the inequality $(1)$ becomes



$$2 cdot frac1lambda Big( frac2lambda^2 - frac1lambda^2 Big)
geq
frac6lambda^3 - frac1lambda cdot frac2lambda^2$$



which reduces to



$$frac2lambda^3 geq frac4lambda^3$$



This is false for all $lambda in (0, infty)$, hence we found a counterexample.






share|cite|improve this answer











$endgroup$












  • $begingroup$
    hi! thanks a lot for this. do you think the inequality is false for a real-valued $X$ (as specified in the question) as well? i haven't been able to find a counter-example. for $X sim mathrmNormal(mu, sigma^2)$, both left and right hand side reduce to $2musigma^2$.
    $endgroup$
    – sk1ll3r
    2 hours ago







  • 1




    $begingroup$
    Sorry I did not notice that detail. I edited my answer to include an example where $X$ has continuous support.
    $endgroup$
    – M. Nestor
    1 hour ago



















2












$begingroup$

Alternate approach / observation...



Assume $2 E[X] Var(X) ge Cov(X, X^2)$ for any real-valued r.v. $X.$ Then in particular it would also be true for $-X$, so that:



$$
beginalign
2E[-X] Var(-X) &ge Cov(-X, (-X)^2)\
-2E[X] Var(X) &ge -Cov(X, X^2)\
2E[X] Var(X) &le Cov(X, X^2)
endalign
$$



which, combined with the original inequality, means: $2E[X]Var(X) = Cov(X, X^2)$ for all $X$.



At this point, it would seem one should be able to find a counter-example easily, without using e.g. $Cov(X, X^2) = E[X^3] - E[X]E[X^2]$ etc, but I just couldn't think of a quick one. :(






share|cite|improve this answer









$endgroup$













    Your Answer








    StackExchange.ready(function()
    var channelOptions =
    tags: "".split(" "),
    id: "69"
    ;
    initTagRenderer("".split(" "), "".split(" "), channelOptions);

    StackExchange.using("externalEditor", function()
    // Have to fire editor after snippets, if snippets enabled
    if (StackExchange.settings.snippets.snippetsEnabled)
    StackExchange.using("snippets", function()
    createEditor();
    );

    else
    createEditor();

    );

    function createEditor()
    StackExchange.prepareEditor(
    heartbeatType: 'answer',
    autoActivateHeartbeat: false,
    convertImagesToLinks: true,
    noModals: true,
    showLowRepImageUploadWarning: true,
    reputationToPostImages: 10,
    bindNavPrevention: true,
    postfix: "",
    imageUploader:
    brandingHtml: "Powered by u003ca class="icon-imgur-white" href="https://imgur.com/"u003eu003c/au003e",
    contentPolicyHtml: "User contributions licensed under u003ca href="https://creativecommons.org/licenses/by-sa/3.0/"u003ecc by-sa 3.0 with attribution requiredu003c/au003e u003ca href="https://stackoverflow.com/legal/content-policy"u003e(content policy)u003c/au003e",
    allowUrls: true
    ,
    noCode: true, onDemand: true,
    discardSelector: ".discard-answer"
    ,immediatelyShowMarkdownHelp:true
    );



    );













    draft saved

    draft discarded


















    StackExchange.ready(
    function ()
    StackExchange.openid.initPostLogin('.new-post-login', 'https%3a%2f%2fmath.stackexchange.com%2fquestions%2f3226379%2fvariance-and-covariance-inequality%23new-answer', 'question_page');

    );

    Post as a guest















    Required, but never shown

























    2 Answers
    2






    active

    oldest

    votes








    2 Answers
    2






    active

    oldest

    votes









    active

    oldest

    votes






    active

    oldest

    votes









    2












    $begingroup$

    Use the variance and covariance identities
    $$textVar(X)=mathbb E[X^2] − mathbb E[X]^2$$
    and
    $$textCov(X,Y) = mathbb E[XY] − mathbb E[X] mathbb E[Y]$$



    Then the given inequality is equivalent to



    $$ 2 mathbb E[X] Big( mathbb E[X^2] - mathbb E[X]^2 Big)
    overset?geq
    mathbb E[X^3] - mathbb E[X] mathbb E[X^2] tag1$$



    For a counterexample, let $X sim textExp(lambda)$ be the exponential distribution with parameter $lambda in (0, infty)$ and probability mass function



    $$ f_X(x) = begincases
    lambda e^-lambda x & textif x geq 0 \
    0 & textif x < 0
    endcases $$



    Its $n$-th moment is given by $mathbb E[X^n]=fracn!lambda^n$, so the inequality $(1)$ becomes



    $$2 cdot frac1lambda Big( frac2lambda^2 - frac1lambda^2 Big)
    geq
    frac6lambda^3 - frac1lambda cdot frac2lambda^2$$



    which reduces to



    $$frac2lambda^3 geq frac4lambda^3$$



    This is false for all $lambda in (0, infty)$, hence we found a counterexample.






    share|cite|improve this answer











    $endgroup$












    • $begingroup$
      hi! thanks a lot for this. do you think the inequality is false for a real-valued $X$ (as specified in the question) as well? i haven't been able to find a counter-example. for $X sim mathrmNormal(mu, sigma^2)$, both left and right hand side reduce to $2musigma^2$.
      $endgroup$
      – sk1ll3r
      2 hours ago







    • 1




      $begingroup$
      Sorry I did not notice that detail. I edited my answer to include an example where $X$ has continuous support.
      $endgroup$
      – M. Nestor
      1 hour ago
















    2












    $begingroup$

    Use the variance and covariance identities
    $$textVar(X)=mathbb E[X^2] − mathbb E[X]^2$$
    and
    $$textCov(X,Y) = mathbb E[XY] − mathbb E[X] mathbb E[Y]$$



    Then the given inequality is equivalent to



    $$ 2 mathbb E[X] Big( mathbb E[X^2] - mathbb E[X]^2 Big)
    overset?geq
    mathbb E[X^3] - mathbb E[X] mathbb E[X^2] tag1$$



    For a counterexample, let $X sim textExp(lambda)$ be the exponential distribution with parameter $lambda in (0, infty)$ and probability mass function



    $$ f_X(x) = begincases
    lambda e^-lambda x & textif x geq 0 \
    0 & textif x < 0
    endcases $$



    Its $n$-th moment is given by $mathbb E[X^n]=fracn!lambda^n$, so the inequality $(1)$ becomes



    $$2 cdot frac1lambda Big( frac2lambda^2 - frac1lambda^2 Big)
    geq
    frac6lambda^3 - frac1lambda cdot frac2lambda^2$$



    which reduces to



    $$frac2lambda^3 geq frac4lambda^3$$



    This is false for all $lambda in (0, infty)$, hence we found a counterexample.






    share|cite|improve this answer











    $endgroup$












    • $begingroup$
      hi! thanks a lot for this. do you think the inequality is false for a real-valued $X$ (as specified in the question) as well? i haven't been able to find a counter-example. for $X sim mathrmNormal(mu, sigma^2)$, both left and right hand side reduce to $2musigma^2$.
      $endgroup$
      – sk1ll3r
      2 hours ago







    • 1




      $begingroup$
      Sorry I did not notice that detail. I edited my answer to include an example where $X$ has continuous support.
      $endgroup$
      – M. Nestor
      1 hour ago














    2












    2








    2





    $begingroup$

    Use the variance and covariance identities
    $$textVar(X)=mathbb E[X^2] − mathbb E[X]^2$$
    and
    $$textCov(X,Y) = mathbb E[XY] − mathbb E[X] mathbb E[Y]$$



    Then the given inequality is equivalent to



    $$ 2 mathbb E[X] Big( mathbb E[X^2] - mathbb E[X]^2 Big)
    overset?geq
    mathbb E[X^3] - mathbb E[X] mathbb E[X^2] tag1$$



    For a counterexample, let $X sim textExp(lambda)$ be the exponential distribution with parameter $lambda in (0, infty)$ and probability mass function



    $$ f_X(x) = begincases
    lambda e^-lambda x & textif x geq 0 \
    0 & textif x < 0
    endcases $$



    Its $n$-th moment is given by $mathbb E[X^n]=fracn!lambda^n$, so the inequality $(1)$ becomes



    $$2 cdot frac1lambda Big( frac2lambda^2 - frac1lambda^2 Big)
    geq
    frac6lambda^3 - frac1lambda cdot frac2lambda^2$$



    which reduces to



    $$frac2lambda^3 geq frac4lambda^3$$



    This is false for all $lambda in (0, infty)$, hence we found a counterexample.






    share|cite|improve this answer











    $endgroup$



    Use the variance and covariance identities
    $$textVar(X)=mathbb E[X^2] − mathbb E[X]^2$$
    and
    $$textCov(X,Y) = mathbb E[XY] − mathbb E[X] mathbb E[Y]$$



    Then the given inequality is equivalent to



    $$ 2 mathbb E[X] Big( mathbb E[X^2] - mathbb E[X]^2 Big)
    overset?geq
    mathbb E[X^3] - mathbb E[X] mathbb E[X^2] tag1$$



    For a counterexample, let $X sim textExp(lambda)$ be the exponential distribution with parameter $lambda in (0, infty)$ and probability mass function



    $$ f_X(x) = begincases
    lambda e^-lambda x & textif x geq 0 \
    0 & textif x < 0
    endcases $$



    Its $n$-th moment is given by $mathbb E[X^n]=fracn!lambda^n$, so the inequality $(1)$ becomes



    $$2 cdot frac1lambda Big( frac2lambda^2 - frac1lambda^2 Big)
    geq
    frac6lambda^3 - frac1lambda cdot frac2lambda^2$$



    which reduces to



    $$frac2lambda^3 geq frac4lambda^3$$



    This is false for all $lambda in (0, infty)$, hence we found a counterexample.







    share|cite|improve this answer














    share|cite|improve this answer



    share|cite|improve this answer








    edited 1 hour ago

























    answered 3 hours ago









    M. NestorM. Nestor

    880115




    880115











    • $begingroup$
      hi! thanks a lot for this. do you think the inequality is false for a real-valued $X$ (as specified in the question) as well? i haven't been able to find a counter-example. for $X sim mathrmNormal(mu, sigma^2)$, both left and right hand side reduce to $2musigma^2$.
      $endgroup$
      – sk1ll3r
      2 hours ago







    • 1




      $begingroup$
      Sorry I did not notice that detail. I edited my answer to include an example where $X$ has continuous support.
      $endgroup$
      – M. Nestor
      1 hour ago

















    • $begingroup$
      hi! thanks a lot for this. do you think the inequality is false for a real-valued $X$ (as specified in the question) as well? i haven't been able to find a counter-example. for $X sim mathrmNormal(mu, sigma^2)$, both left and right hand side reduce to $2musigma^2$.
      $endgroup$
      – sk1ll3r
      2 hours ago







    • 1




      $begingroup$
      Sorry I did not notice that detail. I edited my answer to include an example where $X$ has continuous support.
      $endgroup$
      – M. Nestor
      1 hour ago
















    $begingroup$
    hi! thanks a lot for this. do you think the inequality is false for a real-valued $X$ (as specified in the question) as well? i haven't been able to find a counter-example. for $X sim mathrmNormal(mu, sigma^2)$, both left and right hand side reduce to $2musigma^2$.
    $endgroup$
    – sk1ll3r
    2 hours ago





    $begingroup$
    hi! thanks a lot for this. do you think the inequality is false for a real-valued $X$ (as specified in the question) as well? i haven't been able to find a counter-example. for $X sim mathrmNormal(mu, sigma^2)$, both left and right hand side reduce to $2musigma^2$.
    $endgroup$
    – sk1ll3r
    2 hours ago





    1




    1




    $begingroup$
    Sorry I did not notice that detail. I edited my answer to include an example where $X$ has continuous support.
    $endgroup$
    – M. Nestor
    1 hour ago





    $begingroup$
    Sorry I did not notice that detail. I edited my answer to include an example where $X$ has continuous support.
    $endgroup$
    – M. Nestor
    1 hour ago












    2












    $begingroup$

    Alternate approach / observation...



    Assume $2 E[X] Var(X) ge Cov(X, X^2)$ for any real-valued r.v. $X.$ Then in particular it would also be true for $-X$, so that:



    $$
    beginalign
    2E[-X] Var(-X) &ge Cov(-X, (-X)^2)\
    -2E[X] Var(X) &ge -Cov(X, X^2)\
    2E[X] Var(X) &le Cov(X, X^2)
    endalign
    $$



    which, combined with the original inequality, means: $2E[X]Var(X) = Cov(X, X^2)$ for all $X$.



    At this point, it would seem one should be able to find a counter-example easily, without using e.g. $Cov(X, X^2) = E[X^3] - E[X]E[X^2]$ etc, but I just couldn't think of a quick one. :(






    share|cite|improve this answer









    $endgroup$

















      2












      $begingroup$

      Alternate approach / observation...



      Assume $2 E[X] Var(X) ge Cov(X, X^2)$ for any real-valued r.v. $X.$ Then in particular it would also be true for $-X$, so that:



      $$
      beginalign
      2E[-X] Var(-X) &ge Cov(-X, (-X)^2)\
      -2E[X] Var(X) &ge -Cov(X, X^2)\
      2E[X] Var(X) &le Cov(X, X^2)
      endalign
      $$



      which, combined with the original inequality, means: $2E[X]Var(X) = Cov(X, X^2)$ for all $X$.



      At this point, it would seem one should be able to find a counter-example easily, without using e.g. $Cov(X, X^2) = E[X^3] - E[X]E[X^2]$ etc, but I just couldn't think of a quick one. :(






      share|cite|improve this answer









      $endgroup$















        2












        2








        2





        $begingroup$

        Alternate approach / observation...



        Assume $2 E[X] Var(X) ge Cov(X, X^2)$ for any real-valued r.v. $X.$ Then in particular it would also be true for $-X$, so that:



        $$
        beginalign
        2E[-X] Var(-X) &ge Cov(-X, (-X)^2)\
        -2E[X] Var(X) &ge -Cov(X, X^2)\
        2E[X] Var(X) &le Cov(X, X^2)
        endalign
        $$



        which, combined with the original inequality, means: $2E[X]Var(X) = Cov(X, X^2)$ for all $X$.



        At this point, it would seem one should be able to find a counter-example easily, without using e.g. $Cov(X, X^2) = E[X^3] - E[X]E[X^2]$ etc, but I just couldn't think of a quick one. :(






        share|cite|improve this answer









        $endgroup$



        Alternate approach / observation...



        Assume $2 E[X] Var(X) ge Cov(X, X^2)$ for any real-valued r.v. $X.$ Then in particular it would also be true for $-X$, so that:



        $$
        beginalign
        2E[-X] Var(-X) &ge Cov(-X, (-X)^2)\
        -2E[X] Var(X) &ge -Cov(X, X^2)\
        2E[X] Var(X) &le Cov(X, X^2)
        endalign
        $$



        which, combined with the original inequality, means: $2E[X]Var(X) = Cov(X, X^2)$ for all $X$.



        At this point, it would seem one should be able to find a counter-example easily, without using e.g. $Cov(X, X^2) = E[X^3] - E[X]E[X^2]$ etc, but I just couldn't think of a quick one. :(







        share|cite|improve this answer












        share|cite|improve this answer



        share|cite|improve this answer










        answered 43 mins ago









        antkamantkam

        4,479415




        4,479415



























            draft saved

            draft discarded
















































            Thanks for contributing an answer to Mathematics Stack Exchange!


            • Please be sure to answer the question. Provide details and share your research!

            But avoid


            • Asking for help, clarification, or responding to other answers.

            • Making statements based on opinion; back them up with references or personal experience.

            Use MathJax to format equations. MathJax reference.


            To learn more, see our tips on writing great answers.




            draft saved


            draft discarded














            StackExchange.ready(
            function ()
            StackExchange.openid.initPostLogin('.new-post-login', 'https%3a%2f%2fmath.stackexchange.com%2fquestions%2f3226379%2fvariance-and-covariance-inequality%23new-answer', 'question_page');

            );

            Post as a guest















            Required, but never shown





















































            Required, but never shown














            Required, but never shown












            Required, but never shown







            Required, but never shown

































            Required, but never shown














            Required, but never shown












            Required, but never shown







            Required, but never shown







            Popular posts from this blog

            ParseJSON using SSJSUsing AMPscript with SSJS ActivitiesHow to resubscribe a user in Marketing cloud using SSJS?Pulling Subscriber Status from Lists using SSJSRetrieving Emails using SSJSProblem in updating DE using SSJSUsing SSJS to send single email in Marketing CloudError adding EmailSendDefinition using SSJS

            Кампала Садржај Географија Географија Историја Становништво Привреда Партнерски градови Референце Спољашње везе Мени за навигацију0°11′ СГШ; 32°20′ ИГД / 0.18° СГШ; 32.34° ИГД / 0.18; 32.340°11′ СГШ; 32°20′ ИГД / 0.18° СГШ; 32.34° ИГД / 0.18; 32.34МедијиПодациЗванични веб-сајту

            Кастелфранко ди Сопра Становништво Референце Спољашње везе Мени за навигацију43°37′18″ СГШ; 11°33′32″ ИГД / 43.62156° СГШ; 11.55885° ИГД / 43.62156; 11.5588543°37′18″ СГШ; 11°33′32″ ИГД / 43.62156° СГШ; 11.55885° ИГД / 43.62156; 11.558853179688„The GeoNames geographical database”„Istituto Nazionale di Statistica”проширитиууWorldCat156923403n850174324558639-1cb14643287r(подаци)